Đến nội dung

gadget nội dung

Có 169 mục bởi gadget (Tìm giới hạn từ 04-05-2020)



Sắp theo                Sắp xếp  

#191740 CHÚC CÁC ANH TRÊN DIỄN ĐÀN NGÀY "FATHER DAY "

Đã gửi bởi gadget on 29-09-2008 - 12:02 trong Góc giao lưu

Em cấm mấy anh vô topic của em SPAM nhá !!!!!!!!!!!!!!!!

Hết ngày rồi mà box này dành cho mem spam cho thoải mái mà em :D gì mà bực mình thế :(
Đúng thật nên xử lý anh Ngọc Béo thật nặng vào, anh ĐC đẹp trai, hào hoa,... thế sao chưa có người yêu được.Vô lý quá :D)



#191739 Về sự cố mới

Đã gửi bởi gadget on 29-09-2008 - 11:59 trong Thông báo tổng quan

Vì vậy mà cần có một cuộc cách mạng để đổi mới diễn đàn.

Chỉ cần mọi người có quyết tâm là được theo em thì nên lấy ý kiến xây dựng diễn đàn của tất cả các mems chứ kô nhất thiết mọi chuyện đều bàn trong box kín dành riêng cho Mods :D
Mọi ý kiến dù hay dở gì cũng đáng được hoan nghênh :D



#191729 Về sự cố mới

Đã gửi bởi gadget on 29-09-2008 - 00:56 trong Thông báo tổng quan

Các bài viết cũng bị mất hết các công thức toán latex rồi anh ạ. Cái này có khắc phục được không anh?

Tạm thời cứ quote lại đọc cái code của Latex cũng được :D bạn nào thành thạo 1 số công thức Latex cơ bản may ra mới đọc được :D)
Bọn em sẽ cố gắng phát triển phần nội dung,.... Các anh cũng cố sửa lại hệ thống diễn đàn đi nhé :(



#191728 Bài hay

Đã gửi bởi gadget on 29-09-2008 - 00:50 trong Bất đẳng thức và cực trị

Cho x,y là 2 số thực TM $x^2+y^2-xy=1$
Tìm min,max của $P= x^4+y^4-3xy$

Đặt $x^{2}+y^{2}=a;xy=b$ thì $a\geq 2b$
Từ giả thiết $a-b=1$ Nên$ b+1 \geq 2b \Rightarrow 1 \geq b$ :D
$P=a^{2}-2b^2-3b$
Thay $a=1+b $với điều kiện
biện luận theo tam thức bậc hai ẩn b là được :D hoặc nếu cần bạn có thể dùng phương pháp đồ thị khi tìm min .max tam thức bậc 2 này.



#191709 Chinese Girls’ Mathematics Olympiad

Đã gửi bởi gadget on 28-09-2008 - 16:51 trong Tài nguyên Olympic toán

Cuốn này mình không có bản text ,mình thấy đề của cuộc thi này cũng bình thường mà :D bạn có thể vào mục tài nguyên (resource) của mathlinks click vào từng năm bên mỗi bài đều có línks solution đó bạn :D
Nếu bạn cần sách để in ra cho dễ học thì thử liên hệ anh Tuân :[email protected] thử xem nhé :(
Chúc bạn thành công.



#191686 Giúp em tý

Đã gửi bởi gadget on 21-09-2008 - 12:05 trong Số học

Em xét dạng tổng quát của các phân số trong tổng là :
$\dfrac{\sqrt{k}+\sqrt{k+1}}{2k+1}$ có dạng $\dfrac{a+b}{a^2+b^2}$
em để ý ở đây a và b rất sát nhau (tức gần bằng nhau) nên có thể dùng BDT quen thuộc $(a+b)^2 \leq2(a^2+b^2$) (dấu = xảy ra khi $a=b$
và chú ý khi nào có các dạng như $\dfrac{1}{\sqrt{k} +\sqrt{k+1}}$ thì em có thể liên hệ tới công thức $\dfrac{1}{\sqrt{k} +\sqrt{k+1}}=\sqrt{k+1}-\sqrt{k}$
Nên:
$\dfrac{\sqrt{k}+\sqrt{k+1}}{2k+1} < \dfrac{2}{\sqrt{k}+\sqrt{k+1}}=2.(\sqrt{k+1}-\sqrt{k})$ (k và k+1 không bằng nhau nên dấu bằng khồng thể xảy ra :D )
Từ đây :
thay vào tổng và em rút gọn thì ta được :
$S < 2(\sqrt{49}-1)=26=12$
12 là cái chặn tối nhất mà anh biết .
Cái số $\dfrac{3}{7}$ không đúng đâu vì rõ ràng $\dfrac{1+\sqrt{2}}{3} =0,8..>\dfrac{3}{7}$ rồi em à .
Thế nhé :D



#191681 Tổ hợp

Đã gửi bởi gadget on 21-09-2008 - 11:38 trong Các dạng toán khác

Chú có thể xem hình như Brasil MO 2005 thì phải search trên mathlinks ấy :D



#191679 Buồn ơi buồn đến bao giờ

Đã gửi bởi gadget on 21-09-2008 - 11:01 trong Các dạng toán khác

Cách giải của anh với bài này như sau :
Biểu diễn :$n=2^m.p_1^{k_1}...p_m^{k_m}$
Do đó hàm tổng các ước $\sigma(n)=(2^{m+1}-1) \dfrac{p_1^{k_1+1}-1}{p_1-1} ... \dfrac{p_m^{k_m+1}-1}{p_1-1}=2^{m+1}p_1^{k_1}..p_m^{k_m}+1 $ là số lẻ
1.Ta chứng minh$ k_i$ đều chẵn
Nếu tồn tại $k_i$ lẻ thì $ k_i+1$ chẵn đương nhiên
$\dfrac{p_i^{k_i+1}-1}{p_i-1} $ chẵn .nên $\sigma(n)$ chẵn trái với giả thiết .
Từ đây ta có được $2^{m+1}a^2 \equiv -1 (mod 2^{m+1}-1)$
$\Leftrightarrow a^2 \equiv -1 (mod 2^{m+1}-1) \Leftrightarrow $ -1 là số chính phương mod $p ( \forall p|2^{m+1}-1)$
Nếu m lẻ thì $3|2^{m+1}-1 $ mà $-1$ kô phải số chính phương mod 3 nên m phải chẵn
Do đó n phải là số chính phương.
2.Ta chứng minh n lẻ..... :D
Giả sử $m>0$ thì $2^{m+1}-1$ có dạng 4k+3
do đó nó phải tồn tại 1 ước nguyên tố p dạng $4k+3$(chứng minh cái này đơn giản nhỉ :D)
(-1) không phải số chính phương mod p với p=4k+3
vô lí :D
vậy m=0
nên n lẻ :D
Vậy n chính phương và lẻ :(



#191678 Số phức với hình học phẳng của Yaglom,Đoàn Quỳnh dịch

Đã gửi bởi gadget on 21-09-2008 - 10:39 trong Tài nguyên Olympic toán

Mình biết khá nhiều bài số phức ứng dụng trong hình học phẳng trên tạp chí Hồng không cũng có 1 bài viết về nó :D mình đọc cách đây khá lâu rồi nên không rõ là tờ số tháng năm nào nữa :D
Bạn có thể lập topic mọi người sẽ đóng góp bài :D
Hix học số phức các cuộc thi như quốc gia,tỉnh đâu có được xài :D VN mình chán thật :((



#191638 Các bạn thi KSTN năm 2008 này vào điểm danh nhé

Đã gửi bởi gadget on 19-09-2008 - 21:00 trong Thi HSG cấp Tỉnh, Thành phố. Olympic 30-4. Đề thi và kiểm tra đội tuyển các cấp.

Chúc mừng các chú nhé :clap
@:Ngọc Béo điều tra được năm này VN có em nào xinh hok :-?Tớ chờ dài cổ hơn 1 năm rồi :lol: buồn cho mình thật



#191635 1 bài toán đẹp

Đã gửi bởi gadget on 19-09-2008 - 20:39 trong Các dạng toán khác

Bài này là đề Taiwan 2001 cũ mèm rồi :D
Dùng phản chứng nhé :
giả sử $(m,n)=1 \Rightarrow (5^m-1;5^n-1)=5^1-1=4 $:lol:
Biểu diễn$ 5^m-1=2^a.p_1^{m_1}...p_k^{m_k}$
từ đây ta có được $\phi(5^m-1)=5^n-1=2^{a-1}p_1^{m_1-1}(p_1-1)....p_k^{m_k-1}(p_k-1)$
Nên $m_i=1$
Đến đây bạn có thể chứng minh được rằng m bắt buộc phải lẻ.
Dễ chứng minh $ 5^m-1$ không thể có dạng lũy thừa của 2 được nên $k\leq1$(tức $5^m-1$ có ước nguyên tố lẻ )
Nếu m chẵn thì $5^m-1$ và $5^n-1$ đều chia hết cho 8 vô lí với :wub:
Với $m$ lẻ $m=2k+1$ ta sẽ có được a=2 .mặt khác do $5^m-1=5^{2k}.5 \equiv 1 (mod p_i)$ nên 5 là số chính phương $mod p_i$
dùng luật tương hỗ Gauss ta đi tới $p_i$ là số chính phương mod 5.
nên $p_i \in \{0;1;-1} (mod 5)$
$p_i$ không thể chia hết cho 5 được và nếu tồn tại i để $5|p_i-1$ thì $p_i-1|5^n-1 $ nên $5|5^n-1$ vô lí .
do đó $p_i \equiv -1 (mod 5)$
$ \Rightarrow 5^m-1 \equiv 4.(-1)^k \equiv (-1)^{k+1} (mod 5) \Rightarrow k$ chẵn.
Với k chẵn :
$5^n-1 =2.(p_1-1)..(p_k-1) \equiv 2(-2)^k $ do k chẵn nên $5^n-1$ đồng dư 2 hoặc -2 mod 5
Vô lí
Vậy ta có đpcm :lol:
Bài này công nhận hay thật :wub:
check hộ lời giải cho mình nhé.Lâu mới có thời gian làm toán thú vị như hôm nay.



#191630 CHÚC CÁC ANH TRÊN DIỄN ĐÀN NGÀY "FATHER DAY "

Đã gửi bởi gadget on 19-09-2008 - 19:35 trong Góc giao lưu

Em tên Cầu nè cái gì cũng có xinh gái,học giỏi chỉ tiếc là đến giờ vẫn chưa yêu anh nào :"> hay anh Bồn yêu em đi :lol:



#191609 Các bạn thi KSTN năm 2008 này vào điểm danh nhé

Đã gửi bởi gadget on 18-09-2008 - 23:20 trong Thi HSG cấp Tỉnh, Thành phố. Olympic 30-4. Đề thi và kiểm tra đội tuyển các cấp.

KSTN thì em hiểu rồi nhưng không biết bên HUE với HUS nó tuyển CN CLC với CNTN như thế nào các anh nhỉ. Năm tới em dự định 1 trong 2 trường này mà chưa biết tiêu chuẩn (ngoài HSG QG, QT) vào lớp TN như thê nào cả!!! :lol:

Tính em chơi bời,đam mê tửu sắc thế học HUE và HUS làm gì cho khổ hả em, sang FTU ấy gái nhiều sang đó đủ đởn chơi bời cho nó sướng :lol:
Sang đó dắt mối cho anh luôn.
Hôho hay đó :wub:



#191607 nhờ các anh giải dùm

Đã gửi bởi gadget on 18-09-2008 - 23:14 trong Các bài toán Đại số khác

Bài 2 có vẻ quen thuộc nhỉ lâu r�#8220;i nên chả nhớ có cách nào ngắn gọn hơn nữa
Thử đi cách trâu bò này nhé
$2^x=y^z+1$
a.$z$ chẵn :$z=2k$
Do y phải là số lẻ nên :$ 2^x -2 =y^{2k}-1 =(y^k-1)(y^k+1)$
do x.1 nên $ 2^x-2$ không chia hết cho 4. mà $(y^k-1)(y^k+1)$ chia hết cho 4.=> z kô thể chẵn.
b.$z$ lẻ
$2^x=(y+1)(1+y+..+y^{z-1})$
Nên $y+1=2^m$ nào đó
và $1+y+..+y^{z-1}=2^n ;n>m $
Mà $1+y+..+y^{z-1}=(1+y)+y^2(1+y)+...+y^{z-3}(1+y)+y^{z-1}$ không chia hết (y+1)
Vô lí vậy ta có đpcm
Bài cuối anh kô hiểu sao cho thêm cái 5 mũ vào làm gì :lol:?



#191598 Bình chọn ảnh bạn gái

Đã gửi bởi gadget on 18-09-2008 - 18:07 trong Góc giao lưu

Hơn em có 2 tuổi >:lol: có nên đặt quyết tâm kô nhỉ >>>:lol:
trông...vãi ^^ :wub: >:wub: :D



#191565 Bình chọn ảnh bạn gái

Đã gửi bởi gadget on 17-09-2008 - 22:12 trong Góc giao lưu

Gái vừa xinh vừa đẹp hiếm lắm :lol:
có bà chi này chắc nhiều người ở đây đều biết :lol:
con gái phải thế này chứ
Uổng phí mình sinh muộn mấy năm ^^
Có ông anh nào tán nổi chị này em vái làm sư phụ tán gái của em
Links blog http://360.yahoo.com...4y5FXJKVw-?cq=1
có ảnh chị ấy trên avatar vả cả theme nữa đó ạ :wub:



#191558 Buồn ơi buồn đến bao giờ

Đã gửi bởi gadget on 17-09-2008 - 19:05 trong Các dạng toán khác

Bài này kô ai giải à :lol:
Mình mới chứng minh được mỗi phần n là số chính phương còn n lẻ thì... :lol: ???



#191549 Một Bài Cũ

Đã gửi bởi gadget on 17-09-2008 - 16:25 trong Các dạng toán khác

Bài này có vẻ dễ xơi nhỉ
Có thể tổng quát với n=3k giác đều :lol:
Với k=1 ta được bài toán quen thuộc tam giác ABC đều với M 1 điểm thuộc cung Bc không chứa A thì MA=MC+mB
xuất phát từ bài toán với trường hợp đơn giản ta đi đến bài toán tổng quát
Giả sử $A_1,A_2,..A_3k $là 3k giác đều
Với $A_i$ bất kì tồn tại 1 cặp $A_j,A_k$ duy nhất sao cho $A_i A_jA_k$ là tam giác đều.
Sử dụng định lí Ploteme thì ta có tồn tại 1 đoạn bằng tổng 2 đoạn ( 3 đoạn $MA_i,MA_k,MA_j$ )
Từ đây ta sẽ xếp được các bộ 3 chia thành 2 nhóm thỏa mãn
chỗ này hơi khó giải thích :lol:
có thể tổng quát với các số $m>n;m+n=3k$ sao cho$ \dfrac{m}{n}\leq \2$



#191277 Bình chọn ảnh bạn gái

Đã gửi bởi gadget on 07-09-2008 - 11:59 trong Góc giao lưu

Zzz...Z ! Em méc chị Tr lên chị xử lý các anh :D)



#191272 Hàm số

Đã gửi bởi gadget on 07-09-2008 - 09:57 trong Thi HSG cấp Tỉnh, Thành phố. Olympic 30-4. Đề thi và kiểm tra đội tuyển các cấp.

khả vi là cái gì vậy các anh ơi

Tương đương với có đạo hàm :D



#191209 Bình chọn ảnh bạn gái

Đã gửi bởi gadget on 05-09-2008 - 09:20 trong Góc giao lưu

cái này thì trông béo quá :D
mà sao mấy cái icon diễn đàn mình nghèo nàn thế kô biết :(
sao các anh admin không sửa lại các icon như của YM ấy :D trông đẹp,dễ nhìn hơn nhiều :D



#191208 Bài dễ

Đã gửi bởi gadget on 05-09-2008 - 09:06 trong Thi HSG cấp Tỉnh, Thành phố. Olympic 30-4. Đề thi và kiểm tra đội tuyển các cấp.

Cho $ f(x): [0,\infty) - > R$ thỏa mãn $f(x) = e^{\sqrt {x}} + e^{ - \sqrt {x}}$
Tính giới hạn của $\lim_{n \to \infty} \lim_{x \to 0, x > 0} f^{(n)}(x)$
Have fun :(



#191187 Giúp mình vài bài :|

Đã gửi bởi gadget on 04-09-2008 - 20:26 trong Các bài toán Lượng giác khác

Bài 1 nhé :(
$(\dfrac{a}{c})^{\dfrac{4}{3}}+(\dfrac{b}{c})^{\dfrac{4}{3}}=1$(1)
Nên $a,b <c$
Từ đó $ \dfrac{a}{b}<1 $ nên $(\dfrac{a}{b})^{\dfrac{4}{3}} >(\dfrac{a}{c}))^2(2)$
Tương tự $ (\dfrac{b}{c})^{\dfrac{4}{3}}>>(\dfrac{b}{c}))^2(3)$
Từ (1),(2),(3) ta có:
$b^2 +a^2 <c^2$ nên tam giác ABC tù



#191183 OLympic Toán Quốc Tế 2008

Đã gửi bởi gadget on 04-09-2008 - 18:55 trong Thi HSG Quốc gia và Quốc tế

Bổ sung thêm ý của bạn Primes là 1 học sinh có thể tham gia các kì thi quốc tế cả nhiều môn trong cùng 1 nàm (đương nhiên ngày thi của các cuộc thi đó không trùng nhau rồi :()



#191170 Dãy tốt nhất

Đã gửi bởi gadget on 04-09-2008 - 11:50 trong Các dạng toán khác

Cho dãy $r_1 = 2$ và $r_n = \prod^{n - 1}_{k = 1} r_i + 1, n \geq 2$. Cmr trong tất cả các tập các số nguyên dương ${a_i}$ thỏa mãn :$\sum^{n}_{k = 1} \dfrac {1}{a_i} < 1,$ thì dãy $ r_1,r_2, ... , r_n$ là dãy có tổng nghịch đảo tiến tới gần 1 hơn cả.
Tức với mọi tập $a_{i}$ thì :
$\sum^{n}_{k = 1} \dfrac {1}{a_i} \leq \sum^{n}_{k = 1} \dfrac {1}{r_i} < 1.$